Verstehen der Dichtematrix für reine Zustände vs. gemischte Zustände

Es gibt also eine Frage in meinem Lehrbuch, die das folgende Szenario erklärt:

Ein Physiker führt zwei Experimente A und B durch, um Quantensysteme in verschiedenen Anfangszuständen herzustellen. In Experiment A verwendet er eine probabilistische Maschine, die ein einzelnes Quantensystem in einem von ihnen präparieren kann N mögliche reine Zustände { | ψ 1 , | ψ 2 , . . . , | ψ N } mit entsprechenden Wahrscheinlichkeiten { | P 1 , | P 2 , . . . , | P N } . Im Versuch B , stattdessen erzeugt er m nicht-wechselwirkende Quantensysteme, von denen jedes in seinem entsprechenden niedrigeren Energiezustand präpariert ist { | ϕ 1 , | ϕ 2 , . . . , | ϕ M } . Lassen ρ A Und ρ B seien die Dichtematrixoperatoren für die experimentell hergestellten Quantenzustände A Und B , bzw.

Und fragt folgendes:

  1. Schreiben Sie für beide einen Ausdruck auf ρ A Und ρ B und leiten Sie einen Ausdruck für den erwarteten Wert von ab Ö A ^ Und Ö B ^ (die hermitischen Operatoren, die eine Observable des Systems A bzw. des k-ten Systems des Systems B beschreiben).

  2. Lassen P J Und P k die den Zuständen zugeordneten Projektoren sein | ψ J Und | ψ k produziert von A . Besprechen Sie, ob das Produkt P J P k verschwindet.

Zu Frage 1 habe ich folgendes für system geschrieben A :

(1) ρ A = | ψ J ψ J |

(2) Ö A ^ = T R ( Ö A ^ ρ A ) = T R ( Ö A ^ | ψ J ψ J | )

Meine Begründung für Gleichung 1 ist, dass System A nur reine Zustände erzeugt und dass nur "ein einziger" Zustand erzeugt wird, also enthält die Dichtematrix nur einen Term (hier repräsentiert j den erzeugten j-ten Zustand). Die Frage erwähnt jedoch ihre Wahrscheinlichkeiten, die ich nicht in meine Antwort aufgenommen habe, da nur ein Zustand generiert wird. Sollen P J der Koeffizient in Gleichung (1) und nicht nur die Zahl 1 sein?

Für System B , ich habe folgendes:

(3) ρ B = 1 M ich | ϕ ich ϕ ich |

Meine Begründung dafür ist, dass Experiment B einen gemischten Zustand beschreibt, der insgesamt m Zustände hervorbringt und somit jeder Zustand eine Wahrscheinlichkeit von hat 1 M . Ich bin mir jedoch nicht sicher, wie ich den erwarteten Wert für das Experiment schreiben soll B .

Für die zweite Frage kann ich das Produkt ausdrücken als

(4) P J P k = | ψ J ψ J | ψ k ψ k |

Aber da bekomme ich keine Informationen bezüglich der Orthogonalität der Zustände für Experimente A , wie soll ich ableiten, ob diese Gleichung verschwindet oder nicht?

Wenn jemand Hinweise oder Einblicke in meine Antworten geben könnte, wäre ich sehr dankbar. Ich bin mir nicht ganz sicher, ob ich die Unterschiede zwischen reinen und gemischten Zuständen richtig verstehe.

Antworten (1)

Jeder reine Zustand | ψ N des Quantensystems ist ein Mehrteilchenzustand, der alle Teilchen des Systems beschreibt. Ihre reinen Zustände sind also alle möglichen Zustände des Systems. Da ist es nicht in einem Zustand vorbereitet | ψ es gibt viele möglichkeiten von zuständen, in denen sich ihre systeme befinden könnten .

Nehmen wir an, Sie haben Ihr System vorbereitet | ψ , dann ist es in einem reinen Zustand. Was Physiker einen reinen Zustand nennen, ist im Wesentlichen die Dichtematrix / der statistische Operator:

ρ ^ = | ψ ψ |

Da Ihnen die Wahrscheinlichkeiten des Zustands gegeben sind, die nur reelle Zahlen sind, können Sie davon ausgehen, dass sich Ihr System in einem gemischten Zustand befindet. Außerdem werden Wahrscheinlichkeiten normalerweise nicht als Kets bezeichnet. Für einen gemischten Zustand schreiben Sie:

ρ ^ = ich P ich | ψ ich ψ ich |

Auch für die Wahrscheinlichkeiten muss gelten:

1 = ich P ich

Für Ihre zweite Frage müssen die Zustände weder orthogonal sein noch ein vollständiges orthonormales System bilden, aber sie müssen normalisiert werden.

Ich hoffe, das hilft dir.

Bearbeiten: Für Frage 1 sollten Sie wahrscheinlich auch in Betracht ziehen, die Spur mit einem vollständigen orthonormalen Zustandssystem zu schreiben, z. B. den Eigenzuständen der Dichtematrix.

Vielen Dank für Ihre Antwort. Ich verstehe die verschiedenen Ausdrücke für einen reinen Zustand und einen gemischten Zustand, aber habe ich in Bezug auf meine Antworten die richtige Matrixdichte für jedes System geschrieben? Meine Verwirrung entsteht beim Versuch zu verstehen, ob ρ A ist rein und ρ B ist gemischt
Ich bin mir da nicht 100% sicher, aber ich würde sagen, dass die Dichtematrix in Experiment A gemischt ist, da sie probabilistisch ist, und dass in Experiment B, da Sie m nicht wechselwirkende Quantenzustände haben, Sie m reine Dichtematrizen haben.